210 Peds Unit 4

Réussis tes devoirs et examens dès maintenant avec Quizwiz!

The mother of a toddler reports to the nurse that the child becomes cyanotic when he cries. Which question by the nurse is most important to ask the mother? A. "Does he squat while he cries?" B. "How long does the cyanosis last?" C. "Is he growing normally?" D. "What was his birth weight?"

A. "Does he squat while he cries?" (The mother is describing a "tet" spell, which is a hallmark sign of tetralogy of Fallot. A child with this condition becomes cyanotic when playing or crying and draws his or her legs up or squats. By doing this, the child lowers his or her pulmonary vascular resistance and relieves the cyanosis. The other questions are important, but will not give information specific to this condition.)

The parents of a child with transposition of the great vessels ask the nurse why the child looks blue. Which response by the nurse is the most appropriate? A. "Her body gets blood that doesn't have much oxygen." B. "Her lungs are underdeveloped and underperfused." C. "She is not able to regulate her temperature and is cold." D. "This is very unusual for this condition, so I'll ask the doctor."

A. "Her body gets blood that doesn't have much oxygen." (In this condition, the aorta arises from the right side of the body, so systemic circulation consists of oxygen-poor blood. The other answers are not appropriate.)

The nurse is talking to a parent of an infant with congestive heart failure about feeding the infant. Which of the following statements about feeding the child is correct? a. "You may need to increase the caloric density of your infant's formula." b. "You should feed your baby every 2 hours." c. "You may need to increase the amount of formula your infant eats with each feeding." d. "You should place a nasal oxygen cannula on your infant during and after each

a. "You may need to increase the caloric density of your infant's formula." (The metabolic rate of infants with congestive heart failure is greater because of poor cardiac function and increased heart and respiratory rates. Their caloric needs are greater than those of the average infants, yet their ability to take in the calories is diminished by their fatigue.)

Which assessment would lead the nurse to suspect that a newborn infant has a ventricular septal defect? a. A loud, harsh murmur with a systolic thrill b. Cyanosis when crying c. Blood pressure higher in the arms than in the legs d. A machinery-like murmur

a. A loud, harsh murmur with a systolic thrill (A loud, harsh murmur combined with a systolic thrill is characteristic of a ventricular septal defect.)

Which congenital cardiac defects cause increased pulmonary blood flow? (Select all that apply.) a. Atrial septal defects (ASDs) b. Tetralogy of Fallot c. Dextroposition of aorta d. Patent ductus arteriosus e. Ventricular septal defects (VSDs)

a. Atrial septal defects (ASDs) d. Patent ductus arteriosus e. Ventricular septal defects (VSDs) (The congenital heart defects that cause increased pulmonary blood flow are ASDs, VSDs, and patent ductus arteriosus.)

What assessment(s) in a child with tetralogy of Fallot would indicate the child is experiencing a paroxysmal hypercyanotic episode? (Select all that apply.) a. Spontaneous cyanosis b. Dyspnea c. Weakness d. Dry cough e. Syncope

a. Spontaneous cyanosis b. Dyspnea c. Weakness e. Syncope (Indicators of a paroxysmal hypercyanotic episode or a "tet" episode are spontaneous cyanosis, dyspnea, weakness, and syncope.)

A child has had a closure device inserted in interventional radiology for an atrial septal defect (ASD). Two hours later the child is pale, tachycardic, and hypotensive. Which action by the nurse takes priority? A. Administer a beta blocker to slow the heart rate down. B. Document findings then notify the health-care provider. C. Increase the rate of the IV fluid administration. D. Prepare the child to return to interventional radiology.

D. Prepare the child to return to interventional radiology. (Complications from insertion of closure devices include bleeding, cardiac tamponade, or migration of the device. The provider needs to be notified stat, and the child prepared to return to the interventional radiology suite. A beta blocker is inappropriate in this setting. The nurse should notify the provider and obtain orders prior to changing IV fluid rates. Documentation needs to be thorough, but should wait until after the provider is notified.)

The nurse explained how to position an infant with tetralogy of Fallot if the infant suddenly becomes cyanotic. Which statement by the father leads the nurse to determine he understood the instructions? a. "If the baby turns blue, I will hold him against my shoulder with his knees bent up toward his chest." b. "If the baby turns blue, I will lay him down on a firm surface with his head lower than the rest of his body." c. "If the baby turns blue, I will immediately put the baby upright in an infant seat." d. "If the baby turns blue, I will put the baby in supine position with his head elevated."

a. "If the baby turns blue, I will hold him against my shoulder with his knees bent up toward his chest." (In the event of a paroxysmal hypercyanotic or "tet" spell, the infant should be placed in a knee-chest position.)

Which of the following defects results in decreased pulmonary blood flow? a. Atrial septal defect b. Tetralogy of Fallot c. Ventricular septal defect d. Patent ductus arteriosus

b. Tetralogy of Fallot (Tetralogy of Fallot results in decreased blood flow to the lungs. The pulmonic stenosis increases the pressure in the right ventricle, causing the blood to go from right to left across the ventricular septal defect.)

A 6-month-old infant is receiving digoxin (Lanoxin). The nurse should notify the practitioner and withhold the medication if the apical pulse is less than _____ beats/min. a. 60 b. 70 c. 90 d. 110

c. 90 (If the 1-minute apical pulse is below 90 to 110 beats/min, the digoxin should not be given to a 6-month-old. 60 beats/min is the cut-off for holding the digoxin dose in an adult. 70 beats/min is the determining heart rate to hold a dose of digoxin for an older child. 110 to 120 beats/min is an acceptable heart rate to administer digoxin to a 6-month-old.)

Which defect results in increased pulmonary blood flow? a. Pulmonic stenosis b. Tricuspid atresia c. Atrial septal defect d. Transposition of the great arteries

c. Atrial septal defect (Atrial septal defect results in increased pulmonary blood flow. Blood flows from the left atrium (higher pressure) into the right atrium (lower pressure) and then to the lungs via the pulmonary artery. Pulmonic stenosis is an obstruction to blood flowing from the ventricles. Tricuspid atresia results in decreased pulmonary blood flow. Transposition of the great arteries results in mixed blood flow.)

Which of the following defects results in increased pulmonary blood flow? a. Pulmonic stenosis b. Tricuspid atresia c. Atrial septal defect d. Transposition of the great arteries

c. Atrial septal defect (Atrial septal defect results in increased pulmonary blood flow. Blood flows from the left atrium (higher pressure) into the right atrium (lower pressure) and then to the lungs via the pulmonary artery.)

The nurse should explain to the parents that their child is receiving Lasix for severe congestive heart failure because it is a/an: diuretic. alpha-blocker. form of digitalis. Angiotensin converting enzyme (ACE) inhibitor.

diuretic. (Furosemide (Lasix) is a loop diuretic used to eliminate excess water and salt to prevent reaccumulation of the fluid.)

The doctor suggests that surgery be performed for patent ductus arteriosus (PDA) to prevent: pulmonary infection. right-to-left shunt of blood. decreased workload on left side of heart. increased pulmonary vascular congestion.

increased pulmonary vascular congestion. (The increased pulmonary vascular congestion is the primary complication. The shunt of blood is left to right. The increased pulmonary vascular congestion is the primary complication. Patent ductus arteriosus (PDA) allows blood to flow from the aorta (high pressure) to the pulmonary artery (low pressure). If the PDA stays open, increased pulmonary congestion can occur.)

A patient with heart failure asks the nurse why two diuretic drugs, furosemide (Lasix) and spironolactone (Aldactone), have been prescribed. What is the nurse's best response? a. "Your prescriber wants you to lose fluids and that's why you are taking two diuretics." b. "Each of these diuretics works in a different way to decrease workload on your heart." c. "Using two diuretics will double the amount of fluid you lose to decrease the work your heart must do." d. "Let me contact your prescriber because patients usually do not take two diuretics at the same time."

b. "Each of these diuretics works in a different way to decrease workload on your heart." (Spironolactone, when used in low doses, blocks the action of aldosterone, which causes the body to lose salt and water. Furosemide works in the loop of Henle to decrease reabsorption of salt and water. Spironolactone is usually prescribed with another diuretic to decrease the volume of fluid in the blood vessels, which reduces the workload of the heart. Additionally when used together, these drugs help the body maintain a more normal blood potassium level.)

The nurse is teaching the parents of a young child with iron deficiency anemia about nutrition. What food would the nurse emphasize as being a rich source of iron? a. An egg white b. Cream of Wheat c. A banana d. A carrot

b. Cream of Wheat (Good nutritional sources of iron include boiled egg yolk, liver, green leafy vegetables, Cream of Wheat, dried fruits, beans, nuts, and whole-grain breads.)

A primigravida has just delivered a healthy infant girl. The nurse is about to administer erythromycin ointment in the infant's eyes when the mother asks, "What is that medicine for?" The nurse responds: "It is an eye ointment to help your baby see you better." "It is to protect your baby from contracting herpes from your vaginal tract." "Erythromycin is given prophylactically to prevent a gonorrheal infection." "This medicine will protect your baby's eyes from drying out over the next few days."

"Erythromycin is given prophylactically to prevent a gonorrheal infection." (With the prophylactic use of erythromycin, the incidence of gonococcal conjunctivitis has declined to less than 0.5%. Eye prophylaxis is administered at or shortly after birth to prevent ophthalmia neonatorum. Erythromycin has no bearing on enhancing vision, is used to prevent an infection caused by gonorrhea, not herpes, and is not used for eye lubrication.)

The nurse is preparing an adolescent for discharge after a cardiac catheterization. Which statement by the adolescent would indicate a need for further teaching? "I should avoid tub baths but may shower." "I have to stay on strict bed rest for 3 days." "I should remove the pressure dressing the day after the procedure." "I may attend school but should avoid exercise for several days."

"I have to stay on strict bed rest for 3 days." (The child does not need to be on strict bed rest for 3 days. Showers are recommended; children should avoid a tub bath. The pressure dressing is removed the day after the catheterization and replaced by an adhesive bandage to keep the area clean. Strenuous activity must be avoided for several days, but the child can return to school.)

Which statement by a parent about a child's conjunctivitis indicates that further teaching is needed? "I'll have separate towels and washcloths for each family member." "I'll notify my doctor if the eye gets redder or the drainage increases." "When the eye drainage improves, we'll stop giving the antibiotic ointment." "After taking the antibiotic for 24 hours, my child can return to school."

"When the eye drainage improves, we'll stop giving the antibiotic ointment." (The antibiotic should be continued for the full prescription. Maintaining separate towels and washcloths will prevent the other family members from acquiring the infection. If the infection proliferates, the physician should be contacted. The child should be kept home from school or day care until the child receives the antibiotic for 24 hours.)

A preschool child is scheduled for an echocardiogram. Parents ask the nurse whether they can hold the child during the procedure. The nurse should answer with which response? "You will be able to hold your child during the procedure." "Your child can be active during the procedure, but can't sit in your lap." "Your child must lie quietly; sometimes a mild sedative is administered before the procedure." "The procedure is invasive so your child will be restrained during the echocardiogram."

"Your child must lie quietly; sometimes a mild sedative is administered before the procedure." (Although an echocardiogram is noninvasive, painless, and associated with no known side effects, it can be stressful for children. The child must lie quietly in the standard echocardiographic positions; crying, nursing, being held, or sitting up often leads to diagnostic errors or omissions. Therefore, infants and young children may need a mild sedative; older children benefit from psychologic preparation for the test. The distraction of a video or movie is often helpful.)

Which child should the nurse document as being anemic? 7-year-old child with a hemoglobin of 11.5 g/dL 3-year-old child with a hemoglobin of 12 g/dL 14-year-old child with a hemoglobin of 10 g/dL 1-year-old child with a hemoglobin of 13 g/dL

14-year-old child with a hemoglobin of 10 g/dL (Anemia is a condition in which the number of red blood cells, or hemoglobin concentration, is reduced below the normal values for age. Anemia is defined as a hemoglobin level below 10 or 11 g/dL. The child with a hemoglobin of 10 g/dL would be considered anemic. The normal hemoglobin for a child after 2 years of age is 11.5 to 15.5 g/dL.)

The parent of a toddler consults the nurse regarding the most appropriate time to have the child immunized with the varicella vaccine. The child was hospitalized at age 14 months and required a blood transfusion. Based on the child's age, the most appropriate time for the child to receive the varicella vaccine is at age _____ months. 22 25 28 30

25 (The varicella vaccine should be delayed up to 11 months after the blood transfusion is received.)

The client has received the varicella vaccine and now requires Ig. The client is 4 years and 10 months old. The most appropriate time for the client to receive Ig is at age: 4 years 11 months. 5 years. 5 years 2 months. 6 years.

5 years. (Ig should be delayed for 2 months after receiving the varicella vaccine.)

A child with severe pertussis has been prescribed erythromycin (Erythrocin) and prednisone (Deltasone). Four days later, the parent informs the clinic nurse the child is sleeping more than usual and is not eating well. Which response by the nurse is the most appropriate? A. "Go to the emergency department for an infection workup." B. "Let her sleep; she is just exhausted from having pertussis." C. "Stop the prednisone for now and see how she does tomorrow." D. "Take her temperature; if it's normal bring her in tomorrow."

A. "Go to the emergency department for an infection workup." (Children with severe pertussis are occasionally prescribed steroids to reduce the severity of the symptoms. Steroids, including prednisone, can mask signs of infection (including temperature). A child who is lethargic and not feeding well should be seen immediately for a sepsis workup. The child is acutely ill and needs evaluation, not sleep or waiting until tomorrow to be seen. The child's temperature may be normal or subnormal due to the immune-suppressive effects of the steroids and is not a reliable indicator of infection.)

The day after attending a large birthday party for a classmate, a child breaks out in a rash characteristic of chickenpox. When counseling the parents, which information is most appropriate? A. "Inform all the parents of children at the party that your child has chickenpox." B. "This disease is spread through respiratory droplets, so don't get too close." C. "We can give your child a dose of varicella zoster immune globulin right away." D. "Your child is only contagious for 3 days after the rash first appears."

A. "Inform all the parents of children at the party that your child has chickenpox." (Children with chickenpox are contagious from 1-2 days prior to the rash erupting until the time when all the lesions have crusted over, usually about 7 days. The parents of this child should inform the other parents about their children's exposure to the disease. The disease is spread via airborne and contact routes. Immune globulin can be used within 72 hours after an exposure in immunosuppressed children.)

The student studying pediatric hematological disorders learns that anemia can occur in several ways, including which of the following? (Select all that apply.) A. Acute or chronic blood loss B. Altered shape of RBCs C. Decreased RBC production D. Increased RBC destruction E. Lack of functional RBCs

A. Acute or chronic blood loss C. Decreased RBC production D. Increased RBC destruction (The three major causes of anemia include increased destruction of RBCs, decreased production of RBCs, and blood loss. Altered shape and function do not cause anemia.)

A parent brings a 2-year-old child to the clinic, reporting that the child has an ear infection. Which assessment information leads the nurse to suspect a diagnosis of bacterial otitis media? (Select all that apply.) A. Acute otalgia B. Dull, throbbing pain C. Fever of 104°F (40°C) D. High-pitched crying E. Poor feeding F. Rubbing the ear

A. Acute otalgia C. Fever of 104°F (40°C) E. Poor feeding F. Rubbing the ear (Manifestations of acute otitis media include otalgia, irritability, otorrhea, fever that may be high, poor feeding, rubbing or pulling at the ear, bulging tympanic membrane, enlarged lymph glands, and visualization of purulent material during an otoscopic exam.)

A nursing student asks the instructor why he was marked off on his care plan when explaining a low hemoglobin level as being caused by "anemia." What response by the instructor is best? A. Anemia is a symptom, not a disease. B. Anemia only refers to a low red blood cell count. C. Hemoglobin and anemia are unrelated. D. The hemoglobin must not be too low.

A. Anemia is a symptom, not a disease. (Anemia is a symptom that can be caused by many disease states. It is not a disease that explains low hemoglobin. The other answers are incorrect.)

A 2-year-old child's hemoglobin is 8.2 g/dL. What action by the nurse is best? A. Ask the parents about activity level. B. Document findings in the chart. C. Notify the provider immediately. D. Schedule a re-draw of blood in 6 months.

A. Ask the parents about activity level. (The normal hemoglobin for a child this age is 10.55-12.7 g/dL, so this child is somewhat anemic. The nurse should assess for other manifestations of anemia, including normal activity level. The findings should be documented, but this is not the only action that the nurse should take. The provider needs to be notified, but it does not have to be done immediately, as this is not an emergency. After a full evaluation, the provider may or may not want to repeat the laboratory work in 6 months.)

A nurse is assessing an infant for the most common type of anemia worldwide. What action by the nurse is most helpful? A. Assess if formula is iron-fortified. B. Determine family history of anemia. C. Look at mucous membranes for pallor. D. Perform range of motion on the hips.

A. Assess if formula is iron-fortified. (The most common type of anemia worldwide is iron-deficiency anemia, which can be caused by ingesting non-iron-fortified formula if the child is not breastfed. This type of anemia is not genetic. Pallor, either of the skin or mucous membranes, would be seen in any type of anemia. Range of motion of the hips or shoulders is an important assessment in sickle cell disease, in which avascular necrosis can occur.)

The pediatric clinic nurse is teaching a parent home-care measures for a 12-year-old child diagnosed with viral pharyngitis. Which information is appropriate for the nurse to include in the teaching session? (Select all that apply.) A. Bedrest for a couple days will help recovery. B. Have the child gargle with warm saline. C. Make sure your child continues to eat. D. Offer fluids in small amounts, frequently. E. Warm liquids are better tolerated than cold.

A. Bedrest for a couple days will help recovery. B. Have the child gargle with warm saline. D. Offer fluids in small amounts, frequently. (Children with pharyngitis will recover faster if they stay in bed while acutely ill. An older child can be taught to gargle with warm saline and to use throat lozenges. Cool, bland liquids are usually tolerated better than warm or hot liquids and solid food. The child should not be forced to eat while it is painful, but should stay hydrated.)

Which information about exercise should the nurse give the parents of a child with heart failure? A. Encourage activity on the days when your child feels well. B. Exercise is not allowed in children who have heart failure. C. Physical activity often makes heart failure worse in children. D. Your child will not be able to participate in contact sports.

A. Encourage activity on the days when your child feels well. (Physical activity is beneficial to patients with heart failure, as it can strengthen the heart muscle, possibly preventing or reducing further exacerbations. However, when the child is not feeling well, he or she should not be pushed into vigorous activity. Even walking can be helpful. Without exercise, the symptoms will worsen. Too rigorous an activity may make symptoms worse. Participating in contact sports will probably not be allowed, but this information is too narrow in scope to be the best answer.)

The nurse is caring for a child who experiences frequent ear infections. The child's mother wants to know why this is occurring. Which anatomical differences in the pediatric patient increase the risk for otitis media? (Select all that apply.) A. Impaired drainage B. Longer, thinner eustachian tubes C. Shorter, horizontal eustachian tubes D. Typical lying-down position of infants E. Underdeveloped cartilage lining

A. Impaired drainage C. Shorter, horizontal eustachian tubes D. Typical lying-down position of infants E. Underdeveloped cartilage lining (The following factors lead to otitis media in children: the eustachian tubes are short, wide, and straight and lie in a horizontal plane; the cartilage lining is undeveloped, making the tubes more distensible; the normally abundant pharyngeal lymphoid tissue readily obstructs the eustachian tube openings in the nasopharynx; immature humoral defense mechanisms increase the risk of infections; and the lying-down position of infants favors the pooling of fluid, such as formula, in the pharyngeal cavity.)

The most appropriate nursing diagnosis for a child with anemia is: Activity Intolerance related to generalized weakness. Decreased Cardiac Output related to abnormal hemoglobin. Risk for Injury related to depressed sensorium. Risk for Injury related to dehydration and abnormal hemoglobin.

Activity Intolerance related to generalized weakness. (The basic pathology in anemia is the decreased oxygen-carrying capacity of the blood. The nurse must assess the child's activity level (response to the physiologic state). The nursing diagnosis would reflect the activity intolerance. In generalized anemia no abnormal hemoglobin may be present. Only at a level of very severe anemia does cardiac output become altered. No decreased sensorium exists until profound anemia occurs. Dehydration and abnormal hemoglobin are not usually part of anemia.)

José is a 4-year-old child scheduled for a cardiac catheterization. Preoperative teaching should be: Directed at his parents because he is too young to understand. Detailed in regard to the actual procedures so he will know what to expect. Done several days before the procedure so that he will be prepared. Adapted to his level of development so that he can understand.

Adapted to his level of development so that he can understand. (Preoperative teaching should always be directed at the child's stage of development. The caregivers also benefit from the same explanations. The parents may ask additional questions, which should be answered, but the child needs to receive the information based on developmental level. This age group does not understand in-depth descriptions. Preschoolers should be prepared close to the time of the cardiac catheterization.)

When teaching the mother of a 9-month-old infant about administering liquid iron preparations, the nurse should include that: They should be given with meals. They should be stopped immediately if nausea and vomiting occur. Adequate dosage will turn the stools a tarry green color. Preparation should be allowed to mix with saliva and bathe the teeth before swallowing.

Adequate dosage will turn the stools a tarry green color. (The nurse should prepare the mother for the anticipated change in the child's stools. If the iron dose is adequate, the stools will become a tarry green color. The lack of the color change may indicate insufficient iron. The iron should be given in two divided doses between meals, when the presence of free hydrochloric acid is greatest. Iron is absorbed best in an acidic environment. Vomiting and diarrhea may occur with iron administration. If these occur, the iron should be given with meals, and the dosage reduced and gradually increased as the child develops tolerance. Liquid preparations of iron stain the teeth. They should be administered through a straw, and the mouth rinsed after administration.)

An important nursing consideration when chest tubes will be removed from a child is to: Explain that it is not painful. Explain that only a Band-Aid will be needed. Administer analgesics before the procedure. Expect bright red drainage for several hours after removal.

Administer analgesics before the procedure. (It is appropriate to prepare the child for the removal of chest tubes with analgesics. Short-acting medications can be used that are administered through an existing intravenous line. It is not a pain-free procedure. A sharp, momentary pain is felt, and this should not be misrepresented to the child. A petroleum gauze/airtight dressing is needed. Little or no drainage should be found on removal.)

An important nursing consideration when suctioning a young child who has had heart surgery is to: Perform suctioning at least every hour. Suction for no longer than 30 seconds at a time. Administer supplemental oxygen before and after suctioning. Expect symptoms of respiratory distress when suctioning.

Administer supplemental oxygen before and after suctioning. (If suctioning is indicated, supplemental oxygen is administered with a manual resuscitation bag before and after the procedure to prevent hypoxia. Suctioning should be done only as indicated, not on a routine basis. The child should be suctioned for no more than 5 seconds at one time. Symptoms of respiratory distress are to be avoided by using the appropriate technique.)

What is the appropriate priority nursing action for the infant with a CHD who has an increased respiratory rate, is sweating, and is not feeding well? Recheck the infant's blood pressure. Withhold oral feeding. Alert the physician. Increase the oxygen rate.

Alert the physician. (These are signs of early congestive heart failure, and the physician should be notified. Although rechecking blood pressure may be indicated, it is not the priority action. Withholding the infant's feeding is an incomplete response to the problem. Increasing oxygen may alleviate symptoms; however, medications such as digoxin and furosemide are necessary to improve heart function and fluid retention. Notifying the physician is the priority nursing action.)

When assessing a child for possible congenital heart defects (CHDs), where should the nurse measure blood pressure? The right arm All four extremities The left arm Both arms while the child is crying

All four extremities (When a CHD is suspected, the blood pressure should be measured in all four extremities while the child is quiet. Discrepancies between upper and lower extremities may indicate cardiac disease. Blood pressure measurements for upper and lower extremities are compared during an assessment for CHDs. Blood pressure measurements when the child is crying are likely to be elevated; thus the readings will be inaccurate.)

The nurse is caring for a school-age girl who has had a cardiac catheterization. The child tells the nurse that her bandage is "too wet." The nurse finds the bandage and bed soaked with blood. The most appropriate initial nursing action is to: Notify the physician. Apply a new bandage with more pressure. Place the child in the Trendelenburg position. Apply direct pressure above the catheterization site.

Apply direct pressure above the catheterization site. (If bleeding occurs, direct continuous pressure is applied 2.5 cm (1 inch) above the percutaneous skin site to localize pressure over the vessel puncture. Notifying the physician and applying a new bandage with more pressure can be done after pressure is applied. The nurse can have someone else notify the physician while the pressure is being maintained. The Trendelenburg position would not be helpful; it would increase the drainage from the lower extremities.)

Nursing interventions for the child after a cardiac catheterization include which of the following? (Select all that apply) Allow ambulation as tolerated. Monitor vital signs every 2 hours. Assess the affected extremity for temperature and color. Check pulses above the catheterization site for equality and symmetry. Remove pressure dressing after 4 hours. Maintain a patent peripheral intravenous catheter until discharge.

Assess the affected extremity for temperature and color. Maintain a patent peripheral intravenous catheter until discharge. (The extremity that was used for access for the cardiac catheterization must be checked for temperature and color. Coolness and blanching may indicate arterial occlusion. The child should have a patent peripheral intravenous line to ensure adequate hydration. Allowing ambulation, monitoring vital signs every 2 hours, checking pulses, and removing the pressure dressing after 4 hours are interventions that do not apply to a child after a cardiac catheterization.)

An infant's parents ask the nurse about preventing otitis media (OM). What should the nurse recommend? Avoid tobacco smoke. Use nasal decongestant. Avoid children with OM. Bottle-feed or breastfeed in supine position.

Avoid tobacco smoke. (Eliminating tobacco smoke from the child's environment is essential for preventing OM and other common childhood illnesses. Nasal decongestants are not useful in preventing OM. Children with uncomplicated OM are not contagious unless they show other upper respiratory infection symptoms. Children should be fed in an upright position to prevent OM.)

A child has been hospitalized with rubeola. Which actions by the nursing staff are most important? (Select all that apply.) A. Administer ordered antibiotics on time. B. Assess the child for Koplik's spots. C. Ensure the room is dark for photophobia. D. Monitor the child for febrile seizures. E. Report the disease to health authorities.

B. Assess the child for Koplik's spots. C. Ensure the room is dark for photophobia. E. Report the disease to health authorities. (Appropriate nursing care for the child with rubeola includes assessing the child's mouth for Koplik's spots, providing comfort for photophobia by darkening the room, and reporting the disease to authorities. Rubeola is a viral disease not treated with antibiotics. Fever is moderate and seizures are not usually seen.)

A 7-year-old child is brought to the clinic with what the parent describes as "tonsillitis." The child has a moderate fever, foul breath, and dysphagia, and occasionally spits up lumps of foul-smelling material. Which medication does the nurse prepare to instruct the parents on based on the child's symptoms? A. Amoxicillin (Amoxil) B. Clindamycin (Cleocin) C. Erythromycin (Erythrocin) D. Tetracycline (Sumycin)

B. Clindamycin (Cleocin) (For acute tonsillitis, penicillin, amoxicillin, or erythromycin are the most commonly prescribed antibiotics. However, with the history of this child spitting up lumps of foul-smelling material, it is more likely he or she has chronic tonsillitis, which is best treated with cephalosporin or clindamycin.)

A premature newborn has a pulse pressure of 33 mm Hg. What action by the nurse takes priority? A. Assess the infant for patent ductus arteriosus. B. Ensure the blood pressure cuff is the right size. C. Increase fluids to 1.5 times the maintenance rate. D. Sedate the baby to prevent fighting the ventilator.

B. Ensure the blood pressure cuff is the right size. (A normal pulse pressure in a premature infant is 15-25 mm Hg. This widened pulse pressure could be indicative of a patent ductus, so the nurse should assess for this condition. However, the first action would be to ensure that the blood pressure cuff is the appropriate size and is calibrated correctly. Increasing IV fluids and sedating the baby are not indicated.)

A child has mild anemia. Parents learn to assess for signs of worsening anemia, including which of the following? (Select all that apply.) A. Bradycardia B. Irritability C. Lethargy D. Pale skin E. Rapid heart rate

B. Irritability C. Lethargy D. Pale skin E. Rapid heart rate (Signs of moderate anemia include decreased activity, lethargy, tachycardia, systolic heart murmur, irritability, fatigue, delayed motor development, hepatomegaly, and congestive heart failure. Pale skin can be seen in both mild and moderate anemia.)

One of the most frequent causes of hypovolemic shock in children is: Myocardial infarction. Anaphylaxis. Blood loss. Congenital heart disease.

Blood loss. (Blood loss and extracellular fluid loss are two of the most frequent causes of hypovolemic shock in children. Myocardial infarction is rare in a child; if it occurred, the resulting shock would be cardiogenic, not hypovolemic. Anaphylaxis results in distributive shock from extreme allergy or hypersensitivity to a foreign substance. Congenital heart disease tends to contribute to hypervolemia, not hypovolemia.)

A nurse is educating the parents of a 5-year-old with bacterial otitis media. Which discharge instruction is most important? A. "Bring her back if she is not better in 1 week." B. "Do not allow your child to swim in the future." C. "Give the full course of antibiotics even if she is better." D. "Ice packs are a good way to manage her ear pain."

C. "Give the full course of antibiotics even if she is better." (Giving the full course of antibiotics is crucial to prevent the development of resistant bacteria. If the child is not showing improvement in 48 to 72 hours, she should be reassessed. Swimming is not prohibited after the infection heals. Heat is a better option for ear pain.)

A child hospitalized with heart failure has extremely high blood pressure. Which medication does the nurse prepare to administer? A. Digoxin (Lanoxin) B. Dobutamine (Dobutrex) C. Enalapril (Vasotec) D. Hydrochlorothiazide (Aquazide)

C. Enalapril (Vasotec) (Enalapril is a calcium-channel blocker that reduces systemic vascular resistance, or afterload. Digoxin and dobutamine are positive inotropic agents. Hydrochlorothiazide is a diuretic.)

A nurse is concerned that a child with pulmonary hypertension (PA) is developing heart failure. Which manifestation would the nurse assess for first? A. Cough B. Dyspnea C. Extremity edema D. Tachycardia

C. Extremity edema (As the pressure in the lungs increases in the child with PA, the right ventricle hypertrophies and will eventually fail. Manifestations of right-sided failure include peripheral edema. Lung manifestations are seen in left-sided heart failure. Tachycardia is nonspecific. Although both sides of the heart can eventually fail, the first signs and symptoms will be of right-sided failure.)

A child hospitalized with heart failure has manifestations related to increased preload. Which drug does the nurse prepare to administer? A. Digoxin (Lanoxin) B. Dopamine (Intropin) C. Furosemide (Lasix) D. Metoprolol (Toprol)

C. Furosemide (Lasix) (Furosemide is a diuretic, used to rid the body of excess fluid, and it is excess fluid that leads to increased preload. Digoxin is often used in heart failure for its positive inotropic actions. Dopamine increases contractility. Metoprolol is a beta blocker, and its major effect is blocking sympathetic nervous system activity.)

A child with pertussis is in the catarrhal stage of the disease. Which assessment findings correlate with this condition? (Select all that apply.) A. Chronic cough lasting weeks B. Intense cough causing vomiting C. Low-grade fever D. Sweating and fatigue after coughing E. Upper respiratory symptoms

C. Low-grade fever E. Upper respiratory symptoms (The catarrhal stage of pertussis is characterized by upper respiratory symptoms and low-grade fever. Sweating, fatigue, and intense coughing with vomiting are characteristic of the paroxysmal stage. A chronic cough lasting weeks is seen in the convalescent stage.)

A 5-month-old child is brought to the pediatric clinic by the parent, who reports the child has had a cough for 4 weeks. When reviewing the child's history, the nurse assesses that the child's last DTaP vaccination was at 2 months of age. Which action by the nurse is the most appropriate? A. Inform the provider that both child and parent need antibiotics. B. Make an appointment for the next vaccination at 6 months. C. Obtain nasal washings for a culture and PCR testing. D. Weigh and measure the child; document all findings in the chart.

C. Obtain nasal washings for a culture and PCR testing. (All children with a chronic cough lasting longer than 3 weeks should have diagnostic testing for pertussis, including nasal washings for culture and PCR tests. This child has the risk factor for pertussis of inadequate immunizations; he should have had a second vaccination at 4 months. The best action by the nurse is to collect the specimens. If positive, both child and caretakers will need antibiotics, as pertussis is highly contagious. The child should receive a booster shot as soon as he is recovered. All children should be weighed and measured and findings documented in the chart.)

The pediatric nurse working in a hospital setting uses both standard precautions and transmission-based precautions for patients. Which patient requires only standard precautions? A. Infectious diarrhea B. Staphylococcal infection C. Tonsillitis D. Tuberculosis

C. Tonsillitis (Transmission-based precautions are intended to prevent the transmission of pathogens from those with infectious diseases. Transmission-based precautions include airborne, droplet (TB), and contact precautions (infectious diarrhea and staph infection). Standard precautions are used on all patients, including those with tonsillitis.)

Which drug is an angiotensin-converting enzyme (ACE) inhibitor? Captopril (Capoten) Spironolactone (Aldactone) Furosemide (Lasix) Chlorothiazide (Diuril)

Captopril (Capoten) (Capoten is an ACE inhibitor. Lasix is a loop diuretic. Aldactone blocks the action of aldosterone. Diuril works on the distal tubules.)

The nurse is assessing a child post-cardiac catheterization. Which complication might the nurse anticipate? Cardiac arrhythmia Congestive heart failure Hypostatic pneumonia Rapidly increasing blood pressure

Cardiac arrhythmia (Because a catheter is introduced into the heart, a risk exists of catheter-induced arrhythmias occurring during the procedure. These are usually transient. Hypostatic pneumonia, congestive heart failure, and rapidly increasing blood pressure are not risks usually associated with cardiac catheterization.)

The most common cause of hearing impairment in children is which of the following? Auditory nerve damage Congenital ear defects Congenital rubella Chronic otitis media

Chronic otitis media (Chronic otitis media is the most common cause of hearing impairment in children. It is essential that appropriate measures be instituted to treat existing infections and prevent recurrences.)

A mother tells the nurse that she is discontinuing breastfeeding her 5-month-old infant. The nurse should recommend that the infant be given: Skim milk. Commercial iron-fortified formula. Whole cow's milk. Commercial formula without iron.

Commercial iron-fortified formula. (For children younger than 1 year, the American Academy of Pediatrics recommends the use of breast milk. If breastfeeding has been discontinued, iron-fortified commercial formula should be used. Cow's milk should not be used in children younger than 12 months. Maternal iron stores are almost depleted by this age; the iron-fortified formula will help prevent the development of iron deficiency anemia.)

What is best described as the inability of the heart to pump an adequate amount of blood to the systemic circulation at normal filling pressures? Pulmonary congestion Congestive heart failure Congenital heart defect Systemic venous congestion

Congestive heart failure (The definition of congestive heart failure is the inability of the heart to pump an adequate amount of blood to the systemic circulation at normal filling pressures to meet the metabolic demands of the body. Pulmonary congestion is an excessive accumulation of fluid in the lungs. Congenital heart defect is a malformation of the heart present at birth. Systemic venous congestion is an excessive accumulation of fluid in the systemic vasculature.)

A child has mild anemia and the parent asks why this makes the child have difficulty concentrating. What response by the nurse is best? A. "All sick children have trouble concentrating." B. "Her anemia makes her too tired to think." C. "She may have another problem with her brain." D. "The brain isn't getting enough oxygen."

D. "The brain isn't getting enough oxygen." (Anemia leads to decreased oxygenation of body tissues, including the brain. A lowered cerebral oxygen concentration can lead to dizziness and difficulty concentrating. Stating that all sick children have this problem is inaccurate and vague. The child may be tired, but this answer is also vague and does not really address the question. Describing the possibility of another medical problem is not warranted at this time.)

The mother of a 5-year-old child calls the clinic to ask if her child has a mild respiratory infection or needs to be seen. Which question by the nurse would elicit the most helpful information? A. "Can your child swallow without pain?" B. "Does the child have a sore throat?" C. "Is your child coughing occasionally?" D. "Was the onset gradual or sudden?"

D. "Was the onset gradual or sudden?" (Common "colds" or nasopharyngitis usually have a gradual onset. A rapid onset would indicate a more serious condition. The other manifestations are commonly seen in this disorder.)

A child had a tonsillectomy this morning. What action by the nurse is most important for the child's safety? A. Avoid giving her red popsicles. B. Limit activity the first night. C. Offer ice cream when awake. D. Position the child on her side.

D. Position the child on her side. (After tonsillectomy, children are placed on their side to facilitate drainage and prevent aspiration. Maintaining a patent airway is the priority. Red-colored foods or fluids are not given, as the nurse may not be able to differentiate between the food and bloody drainage; however, this is not a priority for safety. Activity should be limited, but this does not take priority over maintaining the airway. Ice cream and other dairy products are not given because they coat the throat and usually cause coughing or throat clearing, which can lead to bleeding.)

A 5-year-old child is brought into the clinic by a parent, who reports the child has a "sore throat." Which assessment finding would require immediate notification to the health-care provider? A. Difficulty swallowing B. Inflamed, red pharynx C. Refusing to eat the last 2 days D. Strawberry-colored tongue

D. Strawberry-colored tongue (A strawberry-red tongue, petechiae on the palate, and a fine red rash on the trunk or abdomen are consistent with pharyngitis caused by Streptococcus A infection, which needs immediate treatment. The other manifestations are seen with viral pharyngitis infections.)

A child is in the pediatric intensive care unit 2 hours after a surgical repair of an atrial septal defect (ASD). Postoperative nursing actions include which of the following? A. Administer pain medication. B. Maintain venous access. C. Monitor chest tube drainage. D. The nurse will take all of these actions.

D. The nurse will take all of these actions. (All actions are important for the child postoperatively following an ASD repair. This child will be on a mechanical ventilator, so airway is the priority. The nurse will suction secretions as needed to maintain a patent airway.)

An accurate description of anemia is: Increased blood viscosity. Depressed hematopoietic system. Presence of abnormal hemoglobin. Decreased oxygen-carrying capacity of blood.

Decreased oxygen-carrying capacity of blood. (Anemia is a condition in which the number of red blood cells or hemoglobin concentration is reduced below the normal values for age. This results in a decreased oxygen-carrying capacity of blood. Increased blood viscosity is usually a function of too many cells or of dehydration, not of anemia. A depressed hematopoietic system or abnormal hemoglobin can contribute to anemia, but the definition depends on the deceased oxygen-carrying capacity of the blood.)

The nurse is conducting discharge teaching about signs and symptoms of heart failure to parents of an infant with a repaired tetralogy of Fallot. Which signs and symptoms should the nurse include (Select all that apply)? Warm flushed extremities Weight loss Decreased urinary output Sweating (inappropriate) Fatigue

Decreased urinary output Sweating (inappropriate) Fatigue (The signs and symptoms of heart failure include decreased urinary output, sweating, and fatigue. Other signs include pale, cool extremities, not warm and flushed, and weight gain, not weight loss.)

A beneficial effect of administering digoxin (Lanoxin) is that it: Decreases edema. Increases heart size. Decreases cardiac output. Increases venous pressure.

Decreases edema. (Digoxin has a rapid onset and is useful in increasing cardiac output, decreasing venous pressure, and as a result decreasing edema. Heart size is decreased by digoxin.)

The nurse should instruct a child to remain completely still during which procedure in which high-frequency sound waves are translated into images by a transducer? Echocardiography Electrocardiography Cardiac catheterization Electrophysiology

Echocardiography (Echocardiography uses high-frequency sound waves. The child must lie completely still. With the improvements in technology, diagnosis can sometimes be made without cardiac catheterization. Electrocardiography is a tracing of the electrical path of the depolarization action of myocardial cells. Cardiac catheterization is an invasive procedure in which a catheter is threaded into the heart. Electrophysiology is an invasive procedure in which catheters with electrodes are used to record the impulses of the heart directly from the conduction system.)

The nurse is caring for a child after heart surgery. What should she or he do if evidence is found of cardiac tamponade? Increase analgesia. Apply warming blankets. Immediately report this to the physician. Encourage the child to cough, turn, and breathe deeply.

Immediately report this to the physician. (If evidence is noted of cardiac tamponade (blood or fluid in the pericardial space constricting the heart), the physician is notified immediately of this life-threatening complication. Increasing analgesia may be done before the physician drains the fluid, but the physician must be notified. Warming blankets are not indicated at this time. Encouraging the child to cough, turn, and breathe deeply should be deferred until after the evaluation by the physician.)

Iron dextran is ordered for a young child with severe iron deficiency anemia. Nursing considerations include: Administering with meals. Administering between meals. Injecting deeply into a large muscle. Massaging injection site for 5 minutes after administration of drug.

Injecting deeply into a large muscle. (Iron dextran is a parenteral form of iron. When administered intramuscularly, it must be injected into a large muscle using the Z-track method. Iron dextran is for intramuscular or intravenous administration; it is not taken orally. The site should not be massaged to prevent leakage, potential irritation, and staining of the skin.)

Which statement is characteristic of acute otitis media (AOM)? The etiology is unknown. Permanent hearing loss often results. It can be treated by intramuscular antibiotics. It is treated with a broad range of antibiotics.

It is treated with a broad range of antibiotics. (Historically AOM has been treated with a range of antibiotics, and it is the most common disorder treated with antibiotics in the ambulatory setting. The etiology of AOM may be bacterial, such as Streptococcus pneumoniae, Haemophilus influenzae, and Moraxella catarrhalis, or a viral agent. Recent concerns about drug-resistant organisms have caused authorities to recommend judicious use of antibiotics and that antibiotics are not required for initial treatment. Permanent hearing loss is not a frequent cause of properly treated AOM. Intramuscular antibiotics are not necessary. Oral amoxicillin is the treatment of choice.)

Which postoperative intervention should be questioned for a child after a cardiac catheterization? Continue intravenous (IV) fluids until the infant is tolerating oral fluids. Check the dressing for bleeding. Assess peripheral circulation on the affected extremity. Keep the affected leg flexed and elevated.

Keep the affected leg flexed and elevated. (The child should be positioned with the affected leg straight for 4 to 6 hours after the procedure. IV fluid administration continues until the child is taking and retaining adequate amounts of oral fluids. The insertion site dressing should be observed frequently for bleeding. The nurse should also look under the child to check for pooled blood. Peripheral perfusion is monitored after catheterization. Distal pulses should be palpable, although they may be weaker than in the contralateral extremity.)

When preparing a school-age child and the family for heart surgery, the nurse should consider: Not showing unfamiliar equipment. Letting child hear the sounds of an electrocardiograph monitor. Avoiding mentioning postoperative discomfort and interventions. Explaining that an endotracheal tube will not be needed if the surgery goes well.

Letting child hear the sounds of an electrocardiograph monitor. (The child and family should be exposed to the sights and sounds of the intensive care unit. All positive, nonfrightening aspects of the environment are emphasized. The child should be shown unfamiliar equipment, and its use should be demonstrated on a doll. Carefully prepare the child for the postoperative experience, including intravenous lines, incision, and endotracheal tube.)

Which statement best explains why iron deficiency anemia is common during toddlerhood? Milk is a poor source of iron. Iron cannot be stored during fetal development. Fetal iron stores are depleted by age 1 month. Dietary iron cannot be started until age 12 months.

Milk is a poor source of iron. (Children between the ages of 12 and 36 months are at risk for anemia because cow's milk is a major component of their diet, and it is a poor source of iron. Iron is stored during fetal development, but the amount stored depends on maternal iron stores. Fetal iron stores are usually depleted by age 5 to 6 months. Dietary iron can be introduced by breastfeeding, iron-fortified formula, and cereals during the first 12 months of life.)

A nurse is assessing a child with an unrepaired ventricular septal defect. Which heart sound does the nurse expect to assess? S₃ S₄ Murmur Physiologic splitting

Murmur (Murmurs are the sounds that are produced in the heart chambers or major arteries from the back-and-forth flow of blood. These are the sounds expected to be heard in a child with a ventricular septal defect because of the abnormal opening between the ventricles. S₃ is a normal heart sound sometimes heard in children. S₄ is rarely heard as a normal heart sound. If heard, medical evaluation is required. Physiologic splitting is the distinction of the two sounds in S₂, which widens on inspiration. It is a significant normal finding.)

A patient is taking the nonsteroidal antiinflammatory drug indomethacin (Indocin) as treatment for pericarditis. The nurse will teach the patient to watch for which adverse effect? Tachycardia Nervousness Nausea and vomiting Dizziness

Nausea and vomiting (Gastrointestinal effects include dyspepsia, heartburn, epigastric distress, nausea, vomiting, anorexia, abdominal pain, and others. See Table 44-3 for the other adverse effects of nonsteroidal antiinflammatory drugs (NSAIDs). The other options are not adverse effects of NSAIDs.)

Which is most important to document about immunizations in the child's health history? Dosage of immunizations received Occurrence of any reaction after an immunization The exact date the immunizations were received Practitioner who administered the immunizations

Occurrence of any reaction after an immunization (The occurrence of any reaction after an immunization was given is the most important to document in a history because of possible future reactions, especially allergic reactions. Exact dosage of the immunization received may not be recorded on the immunization record. Exact dates are important to obtain but not as important as a history of reaction to an immunization. The practitioner who administered the immunization does not need to be recorded in the health history. A potentially severe physiologic response is the most threatening and most important information to document for safety reasons.)

The nurse is caring for an infant with congestive heart disease (CHD). The nurse should plan which intervention to decrease cardiac demands? Organize nursing activities to allow for uninterrupted sleep. Allow the infant to sleep through feedings during the night. Wait for the infant to cry to show definite signs of hunger. Discourage parents from rocking the infant

Organize nursing activities to allow for uninterrupted sleep. (The infant requires rest and conservation of energy for feeding. Every effort is made to organize nursing activities to allow for uninterrupted periods of sleep. Whenever possible, parents are encouraged to stay with their infant to provide the holding, rocking, and cuddling that help children sleep more soundly. To minimize disturbing the infant, changing bed linens and complete bathing are done only when necessary. Feeding is planned to accommodate the infant's sleep and wake patterns. The child is fed at the first sign of hunger, such as when sucking on fists, rather than waiting until he or she cries for a bottle because the stress of crying exhausts the limited energy supply. Because infants with CHD tire easily and may sleep through feedings, smaller feedings every 3 hours may be helpful.)

The nurse assessing a premature newborn infant auscultates a continuous machinery-like murmur. This finding is associated with which congenital heart defect? Pulmonary stenosis Ventricular septal defect Patent ductus arteriosus Coarctation of the aorta

Patent ductus arteriosus (The classic murmur associated with patent ductus arteriosus is a machinery-like one that can be heard throughout both systole and diastole. A systolic ejection murmur that may be accompanied by a palpable thrill is a manifestation of pulmonary stenosis. The characteristic murmur associated with ventricular septal defect is a loud, harsh, holosystolic murmur. A systolic murmur that is accompanied by an ejection click may be heard on auscultation when coarctation of the aorta is present.)

A clinical manifestation of the systemic venous congestion that can occur with congestive heart failure is: Tachypnea. Peripheral edema. Tachycardia. Pale, cool extremities.

Peripheral edema. (Peripheral edema, especially periorbital edema, is a clinical manifestation of systemic venous congestion. Tachypnea is a manifestation of pulmonary congestion. Tachycardia and pale, cool extremities are clinical manifestations of impaired myocardial function.)

An infant with an unrepaired tetralogy of Fallot defect is becoming extremely cyanotic during a routine blood draw. Which interventions should the nurse implement? List in order from the highest-priority intervention to the lowest-priority intervention. Administer 100% oxygen by blow-by. Place infant in knee-chest position. Remain calm. Give morphine subcutaneously or by an existing intravenous line.

Place infant in knee-chest position. Administer 100% oxygen by blow-by. Give morphine subcutaneously or by an existing intravenous line. Remain calm. (Hypercyanotic spells, also referred to as blue spells or tet spells because they are often seen in infants with tetralogy of Fallot, may occur in any child whose heart defect includes obstruction to pulmonary blood flow and communication between the ventricles. The infant becomes acutely cyanotic and hyperpneic because sudden infundibular spasm decreases pulmonary blood flow and increases right-to-left shunting. Because profound hypoxemia causes cerebral hypoxia, hypercyanotic spells require prompt assessment and treatment to prevent brain damage or possibly death. The infant should first be placed in the knee-chest position to reduce blood returning to the heart. Next, 100% oxygen is given to alleviate the hypoxemia. Morphine is next administered to reduce infundibular spasms. Last, the nurse should remain calm.)

The nurse is providing education to a parent of a 10-month-old infant receiving iron supplements. What will be included in the teaching? Select all that apply. Administer iron with meals. Place iron toward the back side of the mouth with a dropper. Mix iron with milk for greater absorption. Report black, tarry stools to health care provider. Apply barrier ointment if needed to buttocks.

Place iron toward the back side of the mouth with a dropper. Apply barrier ointment if needed to buttocks. (Administration of Iron Supplements includes:Ideally iron supplements should be administered between meals for greater absorption.Liquid iron supplements may stain the teeth, therefore administer with a dropper toward the back of the mouth (side). In older children, administer liquid iron supplements through a straw or rinse mouth thoroughly after ingestion.Avoid administration of liquid iron supplements with whole cow's milk or milk products as these bind free iron and prevent absorption.Educate parents that iron supplements will turn stools black or tarry green.Iron supplements may cause transient constipation. Caution parents not to switch to a low-iron containing formula or whole milk, which are poor sources of iron and may lead to iron deficiency anemia (see Iron Deficiency Anemia, Chapter 43).In older children, follow liquid iron supplement with a citrus fruit or juice drink (no more than 3 to 4 ounces).Avoid administration of iron supplements with food or drinks that bind iron and prevent absorption.)

An 8-month-old infant has a hypercyanotic spell while blood is being drawn. The nurse's first action should be to: Assess for neurologic defects. Place the child in the knee-chest position. Begin cardiopulmonary resuscitation. Prepare the family for imminent death.

Place the child in the knee-chest position. (The first action is to place the infant in the knee-chest position. Blow-by oxygen may be indicated. Neurologic defects are unlikely. The child should be assessed for airway, breathing, and circulation. Often calming the child and administering oxygen and morphine can alleviate the hypercyanotic spell; cardiopulmonary resuscitation is not necessary, and death is unlikely.)

As part of the treatment for congestive heart failure, the child takes the diuretic furosemide. As part of teaching home care, the nurse encourages the family to give the child foods such as bananas, oranges, and leafy vegetables. These foods are recommended because they are high in: Chlorides. Sodium. Potassium. Vitamins.

Potassium. (Diuretics that work on the proximal and distal renal tubules contribute to increased losses of potassium. The child's diet should be supplemented with potassium.)

Surgical closure of the ductus arteriosus would: Stop the loss of unoxygenated blood to the systemic circulation. Decrease the edema in legs and feet. Increase the oxygenation of blood. Prevent the return of oxygenated blood to the lungs.

Prevent the return of oxygenated blood to the lungs. (The ductus arteriosus allows blood to flow from the higher-pressure aorta to the lower-pressure pulmonary artery, causing a right-to-left shunt. If this is surgically closed, no additional oxygenated blood (from the aorta) will return to the lungs through the pulmonary artery. The aorta carries oxygenated blood to the systemic circulation. Because of the higher pressure in the aorta, blood is shunted into the pulmonary artery and the pulmonary circulation. Edema in the legs and feet is usually a sign of heart failure. This repair would not directly affect the edema. Increasing the oxygenation of blood would not interfere with the return of oxygenated blood to the lungs.)

The nurse is caring for a child with persistent hypoxia secondary to a cardiac defect. The nurse recognizes that a risk of cerebrovascular accidents (strokes) exists. An important objective to decrease this risk is to: Minimize seizures. Promote cardiac output. Prevent dehydration. Reduce energy expenditure.

Promote cardiac output. (In children with persistent hypoxia, polycythemia develops. Dehydration must be prevented in hypoxemic children because it potentiates the risk of strokes. Minimizing seizures, promoting cardiac output, and reducing energy expenditure will not reduce the risk of cerebrovascular accidents.)

Which of the following structural defects constitute tetralogy of Fallot? Pulmonic stenosis, ventricular septal defect, overriding aorta, right ventricular hypertrophy Aortic stenosis, ventricular septal defect, overriding aorta, right ventricular hypertrophy Aortic stenosis, atrial septal defect, overriding aorta, left ventricular hypertrophy Pulmonic stenosis, ventricular septal defect, aortic hypertrophy, left ventricular

Pulmonic stenosis, ventricular septal defect, overriding aorta, right ventricular hypertrophy (Tetralogy of Fallot has these four characteristics: pulmonic stenosis, ventricular septal defect, overriding aorta, and right ventricular hypertrophy.)

The nurse is planning activity for a 4-year-old child with anemia. Which activity should the nurse plan for this child? Game of "hide and seek" in the children's outdoor play area Participation in dance activities in the playroom Puppet play in the child's room A walk down to the hospital lobby

Puppet play in the child's room (Because the basic pathologic process in anemia is a decrease in oxygen-carrying capacity, an important nursing responsibility is to assess the child's energy level and minimize excess demands. The child's level of tolerance for activities of daily living and play is assessed, and adjustments are made to allow as much self-care as possible without undue exertion. Puppet play in the child's room would not be overly tiring. Hide and seek, dancing, and walking to the lobby would not conserve the anemic child's energy.)

The leading cause of death after heart transplantation is: Infection. Cardiomyopathy. Rejection. Congestive heart failure.

Rejection. (The posttransplantation course is complex. The leading cause of death after cardiac transplantation is rejection. Infection is a continued risk secondary to the immunosuppression necessary to prevent rejection. Cardiomyopathy is one of the indications for cardiac transplant. Congestive heart failure is not a leading cause of death.)

Seventy-two hours after cardiac surgery, a young child has a temperature of 37.7° C (101° F). The nurse should: Keep the child warm with blankets. Apply a hypothermia blanket. Record the temperature on nurses' notes. Report findings to physician.

Report findings to physician. (In the first 24 to 48 hours after surgery, the body temperature may increase to 37.7° C (100° F) as part of the inflammatory response to tissue trauma. If the temperature is higher or an elevated temperature continues after this period, it is most likely a sign of an infection and immediate investigation is indicated. Blankets should be removed from the child to keep the temperature from increasing. A hypothermia blanket is not indicated for this level of temperature. The temperature should be recorded, but the physician must be notified for evaluation.)

Which factor is most important in predisposing toddlers to frequent infections such as otitis media, tonsillitis, and upper respiratory tract infections? Respirations are abdominal. Pulse and respiratory rates are slower than those in infancy. Defense mechanisms are less efficient than those during infancy. Short, straight internal ear/throat structures and large tonsil/adenoid lymph tissue are present.

Short, straight internal ear/throat structures and large tonsil/adenoid lymph tissue are present. (Toddlers continue to have the short, straight internal ear canal of infants. The lymphoid tissue of the tonsils and adenoids continues to be relatively large. These two anatomic conditions combine to predispose the toddler to frequent infections. The abdominal respirations and lowered pulse and respiratory rate of toddlers do not affect their susceptibility to infection. The defense mechanisms are more efficient compared with those of infancy.)

A nurse is conducting discharge teaching to parents about the care of their infant after cardiac surgery. The nurse instructs the parents to notify the physician if what conditions occur? (Select all that apply) Respiratory rate of 36 at rest Appetite slowly increasing Temperature above 37.7° C (100° F) New, frequent coughing Turning blue or bluer than normal

Temperature above 37.7° C (100° F) New, frequent coughing Turning blue or bluer than normal (The parents should be instructed to notify the physician after their infant's cardiac surgery for a temperature above 37.7° C (100° F); new, frequent coughing; and any episodes of the infant turning blue or bluer than normal. A respiratory rate of 36 at rest for an infant is within normal expectations, and it is expected that the appetite will increase slowly.)

A nurse is teaching nursing students the physiology of congenital heart defects. Which defect results in decreased pulmonary blood flow? Atrial septal defect Ventricular septal defect Tetralogy of Fallot Patent ductus arteriosus

Tetralogy of Fallot (Tetralogy of Fallot results in decreased blood flow to the lungs. The pulmonic stenosis increases the pressure in the right ventricle, causing the blood to go from right to left across the interventricular septal defect. Atrial and ventricular septal defects and patent ductus arteriosus result in increased pulmonary blood flow.)

Parents of a 3-year-old child with congenital heart disease are afraid to let their child play with other children because of possible overexertion. The nurse's reply should be based on knowing that: The child needs opportunities to play with peers. The child needs to understand that peers' activities are too strenuous. Parents can meet all the child's needs. Constant parental supervision is needed to avoid overexertion.

The child needs opportunities to play with peers. (The child needs opportunities for social development. Children usually limit their activities if allowed to set their own pace and regulate their activities. The child will limit activities as necessary. Parents must be encouraged to seek appropriate social activities for the child, especially before kindergarten. The child needs to have activities that foster independence.)

A nurse is teaching a young pregnant woman about vaccines. Which statement is true about vaccines for pregnant women? The varicella vaccine is contraindicated for pregnant women. The rubella vaccine is allowed for pregnant women. The pneumococcal vaccine is contraindicated only for women with asthma. The influenza vaccine is contraindicated for pregnant women.

The varicella vaccine is contraindicated for pregnant women. (The varicella and rubella vaccines are contraindicated. The influenza vaccine is allowed for clients who need it (with asthma). The pneumococcal vaccine should be given to women with asthma or other chronic illness.)

Several blood tests are ordered for a preschool child with severe anemia. She is crying and upset because she remembers the venipuncture done at the clinic 2 days ago. The nurse should explain that: Venipuncture discomfort is very brief. Only one venipuncture will be needed. Topical application of local anesthetic can eliminate venipuncture pain. Most blood tests on children require only a finger puncture because a small amount of blood is needed.

Topical application of local anesthetic can eliminate venipuncture pain. (Preschool children are very concerned about both pain and the loss of blood. When preparing the child for venipuncture, a topical anesthetic will be used to eliminate any pain. This is a very traumatic experience for preschool children. They are concerned about their bodily integrity. A local anesthetic should be used, and a bandage should be applied to maintain bodily integrity. A promise that only one venipuncture will be needed should not be made in case multiple attempts are required. Both finger punctures and venipunctures are traumatic for children. Both require preparation.)

Which is considered a mixed cardiac defect? Pulmonic stenosis Atrial septal defect Patent ductus arteriosus Transposition of the great arteries

Transposition of the great arteries (Pulmonic stenosis is classified as an obstructive defect. Atrial septal defect is classified as a defect with increased pulmonary blood flow. Patent ductus arteriosus is classified as a defect with increased pulmonary blood flow. Transposition of the great arteries allows the mixing of blood in the heart.)

In which situation is there the greatest risk that a newborn infant will have a congenital heart defect (CHD)? Trisomy 21 detected on amniocentesis Family history of myocardial infarction Father has type 1 diabetes mellitus Older sibling born with Turner's syndrome

Trisomy 21 detected on amniocentesis (The incidence of congenital heart disease is approximately 50% in children with trisomy 21 (Down syndrome). A family history of congenital heart disease, not acquired heart disease, increases the risk of giving birth to a child with CHD. Infants born to mothers who are insulin dependent have an increased risk of CHD. Infants identified as having certain genetic defects, such as Turner's syndrome, have a higher incidence of CHD.)

Which intervention should be included in the plan of care for an infant with the nursing diagnosis of Excess Fluid Volume related to congestive heart failure? Weigh the infant every day on the same scale at the same time. Notify the physician when weight gain exceeds more than 20 g/day. Put the infant in a car seat to minimize movement. Administer digoxin (Lanoxin) as ordered by the physician.

Weigh the infant every day on the same scale at the same time. (Excess fluid volume may not be overtly visible. Weight changes may indicate fluid retention. Weighing the infant on the same scale at the same time each day ensures consistency. An excessive weight gain for an infant is an increase of more than 50 g/day. With fluid volume excess, skin will be edematous. The infant's position should be changed frequently to prevent undesirable pooling of fluid in certain areas. Lanoxin is used in the treatment of congestive heart failure to improve cardiac function. Diuretics will help the body get rid of excess fluid.)

Which statement by a mother may indicate a cause for her 9-month-old's iron deficiency anemia? a. "Formula is so expensive. We switched to regular milk right away." b. "She almost never drinks water." c. "She doesn't really like peaches or pears, so we stick to bananas for fruit." d. "I give her a piece of bread now and then. She likes to chew on it."

a. "Formula is so expensive. We switched to regular milk right away." (Because cow's milk contains very little iron, infants should drink iron-fortified formula for the first year of life.)

Recent studies indicate that a deficiency of which of the following vitamins correlates with increased morbidity and mortality in children with measles? a. A b. C c. Niacin d. Folic acid

a. A (Vitamin A deficiency is correlated with increased morbidity and mortality in children with measles. This vitamin deficiency also is associated with complications from diarrhea, and infections are often increased in infants and children with vitamin A deficiency.)

What is the best choice for fluid replacement that the nurse can offer a child who has just had a tonsillectomy? a. A popsicle b. Chocolate milk c. Orange juice d. Cola drink

a. A popsicle (Small amounts of clear liquids can be offered to the child. Synthetic fruit juices are not as irritating as natural juices. A popsicle is usually well-tolerated.)

A 4-year-old child is receiving amoxicillin (Amoxil) to treat otitis media and is in the clinic for a well-child checkup on the last day of antibiotic therapy. The provider orders varicella (Varivax); mumps, measles, and rubella (MMR); inactivated polio (IPV); and diphtheria, tetanus, and acellular pertussis (DTaP) vaccines to be given. Which action by the nurse is correct? a. Administer the vaccines as ordered. b. Discuss giving the MMR vaccine in 4 weeks. c. Hold all vaccines until 2 weeks after antibiotic therapy. d. Recommend aspirin for fever and discomfort.

a. Administer the vaccines as ordered. (Antibiotic therapy is not generally a contraindication to the use of vaccines. Vaccines may be given in cases of mild acute illness or during the convalescent phase of an illness. All four vaccines may be given. If the MMR or other live virus vaccine is not given the same day as the varicella vaccine, administration of the two vaccines should be separated by at least 4 weeks. Aspirin should not be given because of the increased risk of Reye's syndrome.)

The nurse would observe a child for frequent swallowing after a tonsillectomy and adenoidectomy (T&A). What might this indicate? a. Bleeding from the surgical site b. Pain at the incision area c. Sore throat from postnasal drip d. Potential vomiting

a. Bleeding from the surgical site (Hemorrhage is the most common postoperative complication. Blood trickling down the back of the child's throat could cause frequent swallowing.)

A 2-month-old infant is in for a well-baby visit. Which of the following immunizations should the nurse administer to the infant? a. DTaP-1 b. MMR c. Varicella d. Influenza

a. DTaP-1 (The Centers for Disease Control and Prevention (2007) recommends the following vaccines at 2 months of age: DTaP-1, Hib-1, PCV, Polio, RV.)

Recommendations for hepatitis B (HBV) vaccine include which of the following? a. First dose is given between birth and age 2 days. b. First dose is given between ages 12 and 15 months. c. It is not recommended for neonates who are at low risk for hepatitis B. d. It is not recommended for neonates whose mothers are positive for HBV.

a. First dose is given between birth and age 2 days. (To reduce the incidence of HBV in children and its serious consequences in adulthood, the first of three doses is recommended soon after birth and before hospital discharge.)

Which of the following is the most common cause of anemia in preterm infants? a. Frequent blood sampling b. Respiratory distress syndrome c. Meconium aspiration syndrome d. Persistent pulmonary hypertension

a. Frequent blood sampling (The most common cause of anemia in preterm infants is frequent blood-sample withdrawal and inadequate erythropoiesis in acutely ill infants. Microsamples should be used for blood tests, and the amount of blood drawn should be monitored.)

A father asks why his child with tetralogy of Fallot seems to favor a squatting position. What is the nurse's best response? a. Squatting increases the return of venous blood back to the heart. b. Squatting decreases arterial blood flow away from the heart. c. Squatting is a common resting position when a child is tachycardic. d. Squatting increases the workload of the heart.

a. Squatting increases the return of venous blood back to the heart. (The squatting position allows the child to breathe more easily because systemic venous return is increased.)

Which of the following statements concerning hearing in school-age children is correct? a. The rate of otitis media infections met Healthy People 2010 targets. b. Hearing and visual problems occur with the same frequency. c. All 50 states require routine hearing evaluations. d. The use of tympanograms has improved the accuracy of hearing screenings.

a. The rate of otitis media infections met Healthy People 2010 targets. (Hearing problems occur with less frequency than do vision problems. Many states, but not all, require routine hearing evaluations. Tympanograms measure how well the tympanic membrane vibrates.)

The most common type of hearing loss, which results from interference of transmission of sound to the middle ear, is called: a. conductive. b. sensorineural. c. mixed conductive-sensorineural. d. central auditory imperceptive.

a. conductive. (Conductive or middle-ear hearing loss is the most common type. It results from interference of transmission of sound to the middle ear, most often from recurrent otitis media.)

It is important that a child with Group A β-hemolytic streptococci (GABHS) infection be treated with antibiotics to prevent: otitis media. diabetes insipidus. nephrotic syndrome. acute rheumatic fever.

acute rheumatic fever. (Otitis media and diabetes insipidus are not sequelae to group A β-hemolytic streptococci (GABHS). Otitis media and diabetes insipidus are not sequelae to GABHS. Children are at risk for glomerulonephritis, not nephritic syndrome. Children with Group A β-hemolytic streptococci (GABHS) infection are at risk for acute rheumatic fever and acute glomerulonephritis.)

The nurse is discussing vaccines with the mother of a 4-year-old child who attends a day care center that requires the DTaP vaccine. The mother, who is pregnant, tells the nurse that she does not want her child to receive the pertussis vaccine because she has heard that the disease is "not that serious" in older children. What information will the nurse include when discussing this with the mother? a. If she gets the vaccine, both she and her 4 year-old child will be protected. b. If the 4-year-old child contracts pertussis, it can be passed on to her newborn. c. The vaccine will not be given to her child while she is pregnant. d. Vaccinating the 4-year-old will provide passive immunity for her unborn child.

b. If the 4-year-old child contracts pertussis, it can be passed on to her newborn. (Even though pertussis is not as serious in older children, it is important to vaccinate children to prevent the spread of the disease to infants and others who are not immunized and who are at risk for significant morbidity and mortality from this disease. Vaccinating the mother will not protect the 4-year-old from getting pertussis. The DTaP vaccine may be given to children whose mothers are pregnant. Vaccinating the child does not confer passive immunity to the unborn child.)

In which of these communicable diseases are Koplik spots present? a. Rubella b. Measles (rubeola) c. Chickenpox (varicella) d. Exanthema subitum (roseola)

b. Measles (rubeola) (Koplik spots are small irregular red spots with a minute, bluish white center found on the buccal mucosa 2 days before systemic rash.)

A 14-year-old adolescent never had chickenpox as a child. Which should the nurse recommend? a. One dose of the varicella vaccination b. Two doses of the varicella vaccination 4 weeks apart c. One dose of the varicella immune globulin d. No vaccinations—the child is past the age to receive it

b. Two doses of the varicella vaccination 4 weeks apart (All adolescents should also be assessed for previous history of varicella infection or vaccination. Vaccination with the varicella vaccine is recommended for those with no previous history; for those with no previous infection or history, the varicella vaccine may be given in two doses 4 or more weeks apart to adolescents 13 years or older. The varicella immune globulin is given to immunosuppressed children exposed to chickenpox to boost immunity; it is only temporary. The varicella vaccination should be given to adolescents, no matter the age, who have not had chickenpox as a child.)

When taking the history of a child hospitalized with Reye syndrome, the nurse should not be surprised that a week ago the child had recovered from which of the following? a. Measles b. Varicella c. Meningitis d. Hepatitis

b. Varicella (Most cases of Reye syndrome follow a common viral illness such as varicella or influenza.)

When is a child with chickenpox considered to be no longer contagious? a. When fever is absent b. When lesions are crusted c. 24 hours after lesions erupt d. 8 days after onset of illness

b. When lesions are crusted (When the lesions are crusted, the chickenpox is no longer contagious. This may be a week after onset of disease.)

What will the nurse administer with ferrous sulfate drops when providing them to a child on the pediatric unit? a. With milk b. With orange juice c. With water d. On a full stomach

b. With orange juice (Vitamin C aids in the absorption of iron, whereas food and milk interfere with the absorption of iron.)

Strict isolation is required for a child who is hospitalized with: a. mumps. b. chickenpox. c. exanthema subitum (roseola). d. erythema infectiosum (fifth disease).

b. chickenpox. (Chickenpox is communicable through direct contact, droplet spread, and contaminated objects.)

Acyclovir (Zovirax) is given to children with chickenpox to: a. minimize scarring. b. decrease the number of lesions. c. prevent aplastic anemia. d. prevent spread of the disease

b. decrease the number of lesions. (Acyclovir decreases the number of lesions; shortens duration of fever; and decreases itching, lethargy, and anorexia.)

A mother tells the nurse she is afraid to have her infant immunized. Which statement would be the most appropriate response for the nurse? a. "It is normal to be concerned, as some immunizations have been linked to autism." b. "Researching the Internet will provide you more information as to the importance of immunizations." c. "Choosing to not vaccinate your child puts your child and others at risk." d. "Your infant received active immunity at birth, so immunizations are not indicated until 4 months of age."

c. "Choosing to not vaccinate your child puts your child and others at risk." (The child who is not immunized and others around the child are at risk if immunizations do not occur. Scientific studies have not found a relationship between immunizations and autism. Information that is researched on the Internet may not be a reliable source of information as the nurse has no control over the information the client receives. Infants receive natural passive immunity at birth through the placental transfer of maternal antibodies; this protection lasts for about 2 months.)

A child's parents have requested that their child be exempt from obtaining the required immunizations for a religious reason. What action should a school nurse take related to this request? a. Deny entry into school for children without the required immunizations. b. Educate the parents about the need for immunizations. c. Be aware of the state laws regarding acceptable reasons for exemption. d. Allow the student to attend school without the immunizations.

c. Be aware of the state laws regarding acceptable reasons for exemption. (The state mandates the rules about having immunizations that the nurse must follow. Thus, it is the responsibility of the school nurse to be aware of the laws in the state regarding acceptable reasons for immunization exemption.)

What will the nurse teach parents when giving instructions for acute conjunctivitis? a. Apply cool compresses to the affected eye several times a day. b. Instill topical steroid eye drops for 1 week. c. Clear drainage from the inner to the outer aspect of the eye. d. Keep the eye patched until the inflammation resolves.

c. Clear drainage from the inner to the outer aspect of the eye. (Eye secretions are always cleared from the inner canthus downward and away from the opposite eye (inner to outer direction).)

Which of the following statements best describes iron deficiency anemia in infants? a. It is caused by depression of the hematopoietic system. b. It is easily diagnosed because of infant's emaciated appearance. c. Clinical manifestations are similar regardless of the cause of the anemia. d. Clinical manifestations result from a decreased intake of milk and the premature addition of solid foods.

c. Clinical manifestations are similar regardless of the cause of the anemia. (In iron deficiency anemia, the child's clinical appearance is a result of the anemia, not the underlying cause.)

Which of the following is best described as the inability of the heart to pump an adequate amount of blood to the systemic circulation at normal filling pressures? a. Pulmonary congestion b. Congenital heart defect c. Congestive heart failure d. Systemic venous congestion

c. Congestive heart failure (The definition of congestive heart failure is the inability of the heart to pump an adequate amount of blood to the systemic circulation at normal filling pressures to meet the body's metabolic demands.)

The single parent of a 12-year-old child who has just been diagnosed with chickenpox tells the nurse that she cannot afford to stay home with the child and miss work. The parent asks the nurse if some medication will shorten the course of the illness. Which of the following is the most appropriate nursing intervention? a. Reassure the parent that it is not necessary to stay home with the child. b. Explain that no medication will shorten the course of the illness. c. Explain the advantages of the medication acyclovir to treat chickenpox. d. Explain the advantages of the medication varicella-zoster immune globulin

c. Explain the advantages of the medication acyclovir to treat chickenpox. (Acyclovir is effective in treating the number of lesions; shortening the duration of fever; and decreasing itching, lethargy, and anorexia.)

Which factor would indicate a medical contraindication for the administration of immunizations? a. Religious beliefs b. Sore throat c. Leukemia d. Fear of side effects

c. Leukemia (Religious beliefs, sore throat, and fear of side effects are not medical contradictions.)

How would the nurse advise a mother to clear the nostrils when her infant has a cold? a. Clear the nasal passages after the infant has a feeding. b. Use over-the-counter nose drops to clear passages. c. Remove nasal secretions with a bulb syringe. d. Instill saline nose drops after clearing away secretions.

c. Remove nasal secretions with a bulb syringe. (The nasal passages can be cleared by instilling a few drops of saline into the nose and then suctioning the secretions with a bulb syringe.)

It is generally recommended that a child with acute streptococcal pharyngitis can return to school: a. when sore throat is better. b. if no complications develop. c. after taking antibiotics for 24 hours. d. after taking antibiotics for 3 days.

c. after taking antibiotics for 24 hours. (After children have taken antibiotics for 24 hours, they are no longer contagious to other children.)

A parent reports to the nurse that her child has inflamed conjunctivae of both eyes with purulent drainage and crusting of the eyelids, especially on awakening. These manifestations suggest: a. viral conjunctivitis. b. allergic conjunctivitis. c. bacterial conjunctivitis. d. conjunctivitis caused by foreign body.

c. bacterial conjunctivitis. (Bacterial conjunctivitis has these symptoms.)

An important nursing responsibility when a dysrhythmia is suspected is to: order an immediate electrocardiogram. count the radial rate at 1-minute intervals 5 times in a row. count the apical rate for 1 full minute and compare it with the radial rate. have someone else take the radial rate while the nurse simultaneously checks the apical rate.

count the apical rate for 1 full minute and compare it with the radial rate. (This may be indicated after conferring with the practitioner. The radial pulse needs to be compared with the apical. This is the nurse's first action. If an arrhythmia is occurring, the radial pulse may be lower than the apical rate. It is the responsibility of the nurse to check both rates.)

Parents have understood teaching about prevention of childhood otitis media if they make which statement? a. "We will only prop the bottle during the daytime feedings." b. "Breastfeeding will be discontinued after 4 months of age." c. "We will place the child flat right after feedings." d. "We will be sure to keep immunizations up to date."

d. "We will be sure to keep immunizations up to date." (Parents have understood the teaching about preventing childhood otitis media if they respond they will keep childhood immunizations up to date. The child should be maintained upright during feedings and after. Otitis media can be prevented by exclusively breastfeeding until at least 6 months of age. Propping bottles is discouraged to avoid pooling of milk while the child is in the supine position.)

Chronic otitis media with effusion (OME) is differentiated from acute otitis media (AOM) because it is usually characterized by which of the following? a. Fever as high as 40° C (104° F) b. Severe pain in the ear c. Nausea and vomiting d. A feeling of fullness in the ear

d. A feeling of fullness in the ear (OME is characterized by feeling of fullness in ear or other nonspecific complaints.)

The nurse is recommending how to prevent iron deficiency anemia in a healthy, term, breast-fed infant. Which should be suggested? a. Iron (ferrous sulfate) drops after age 1 month b. Iron-fortified commercial formula by age 4 to 6 months c. Iron-fortified infant cereal by age 2 months d. Iron-fortified infant cereal by age 4 to 6 months

d. Iron-fortified infant cereal by age 4 to 6 months (Breast milk supplies inadequate iron for growth and development after age 5 months. Supplementation is necessary at this time. The mother can supplement the breastfeeding with iron-fortified infant cereal. Iron supplementation or the introduction of solid foods in a breast-fed baby is not indicated. Providing iron-fortified commercial formula by age 4 to 6 months should be done only if the mother is choosing to discontinue breastfeeding.)

Which of the following statements is true concerning susceptibility to pertussis? a. Neonates will be immune the first few months. b. If mother has had the disease, infant will receive passive immunity. c. Children younger than 1 year seldom contract this disease. d. Most children are highly susceptible from birth.

d. Most children are highly susceptible from birth. (The acellular pertussis vaccine is recommended by the American Academy of Pediatrics beginning at age 6 weeks. Infants are at greater risk for complications of pertussis. The vaccine is not given after age 7 years, when the risks of the vaccine become greater than those of pertussis.)

A young adult patient is in the clinic to receive a tetanus vaccine after sustaining a laceration injury. The nurse learns that the patient, who works in a day care center, has not had any vaccines for more than 10 years. Which vaccine will the nurse expect to administer? a. DT b. DTaP c. Td d. Tdap

d. Tdap (Persons who work with children should receive acellular pertussis vaccine. The Tdap is given to adults. The DTaP is given to children up to age 6. The DT and Td do not contain pertussis.)

A parent comments that her infant has had several ear infections in the past few months. Why are infants more susceptible to otitis media? a. Infants are in a supine or prone position most of the time. b. Sucking on a nipple creates middle ear pressure. c. They have increased susceptibility to upper respiratory tract infections. d. The eustachian tube is short, straight, and wide.

d. The eustachian tube is short, straight, and wide. (An infant's eustachian tubes are short, wide, and straight, allowing microorganisms easy access to the middle ear.)

A patient who has heart failure receives digoxin (Lanoxin) and an angiotension-converting enzyme (ACE) inhibitor. The patient will begin taking spironolactone (Aldactone). The patient asks why the new drug is necessary. The nurse will tell the patient that spironolactone will be given for which reason? a. To enhance potassium excretion b. To increase cardiac contractility c. To minimize fluid losses d. To provide cardioprotective effects

d. To provide cardioprotective effects (Spironolactone is a potassium-sparing diuretic that blocks production of aldosterone, causing improved heart rate variability and decreased myocardial fibrosis. It is given in congestive heart failure for its cardioprotective effects. Spironolactone does not directly alter cardiac contractility but may slightly decrease contractility if fluid volume is decreased. It is a mild diuretic but is not given in this instance to minimize fluid losses.)

Which of the following may be given to high-risk children after exposure to chickenpox to prevent varicella? a. Acyclovir b. Varicella globulin c. Diphenhydramine hydrochloride d. VZIG

d. VZIG (VZIG is given to high-risk children to prevent the development of chickenpox.)

The nurse is documenting a description of a skin assessment. What term can be used for an elevated, fluid-filled blister? a. Pustule b. Papule c. Wheal d. Vesicle

d. Vesicle (A vesicle is an elevated, fluid-filled blister (cold sore, chickenpox).)

A mother tells the nurse that she does not want her infant immunized because of the discomfort associated with injections. The nurse should explain that: a. this cannot be prevented. b. infants do not feel pain as adults do. c. this is not a good reason for refusing immunizations. d. a topical anesthetic, EMLA, can be applied before injections are given.

d. a topical anesthetic, EMLA, can be applied before injections are given. (Several topical anesthetic agents can be used to minimize the discomfort associated with immunization injections. These include EMLA (eutectic mixture of local anesthetic) and vapor coolant sprays.)

The parent of an infant with nasopharyngitis should be instructed to notify the health professional if the infant: a. becomes fussy. b. has a cough. c. has a fever over 99° F. d. shows signs of an earache.

d. shows signs of an earache. (If an infant with nasopharyngitis shows signs of an earache, it may mean a secondary bacterial infection is present and the infant should be referred to a practitioner for evaluation. Irritability is common in an infant with a viral illness. Cough can be a sign of nasopharyngitis. Fever is common in viral illnesses.)

Nurses counseling parents regarding the home care of the child with a cardiac defect before corrective surgery should stress the: importance of reducing caloric intake to decrease cardiac demands. importance of relaxing discipline and limit-setting to prevent crying. need to be extremely concerned about cyanotic spells. desirability of promoting normalcy within the limits of the child's condition.

desirability of promoting normalcy within the limits of the child's condition. (Child needs increased caloric intake. Child needs discipline and appropriate limits. Because cyanotic spells occur in children with some defects, the parents need to be taught how to manage these. The child needs to have social interactions, discipline, and appropriate limit-setting. Parents need to be encouraged to promote as normal a life as possible for their child.)

An appropriate intervention to provide comfort for the child with itching associated with chickenpox is to: encourage frequent warm baths. give aspirin or acetaminophen. give an antipruritic medication such as Benadryl. apply thick coat of Caladryl lotion over open lesions.

give an antipruritic medication such as Benadryl. (Cool baths are recommended for relief of itching. Neither drug has antiitching effects. Antipruritic medicines such as diphenhydramine (Benadryl) are useful for severe itching, which interferes with sleep and may contribute to secondary infection. Caladryl lotion (contains Benadryl) should be applied sparingly over open lesions to minimize absorption.)

The parent of a child receiving an iron preparation tells the nurse that the child's stools are a tarry green color. The nurse should explain that this is a/an: symptom of iron-deficiency anemia. adverse effect of the iron preparation. indicator of an iron preparation overdose. normally expected change caused by the iron preparation.

normally expected change caused by the iron preparation. (These descriptions are not relevant. If the stools do not become a tarry green or black color, it may indicate administration issues. An adequate dosage of iron turns the stools a tarry green or black color.)

The nurse is preparing to give digoxin to a 9-month-old infant. He or she checks the dose and draws up 4 mL of the drug. The MOST appropriate nursing action is to: not give the dose; suspect dosage error. mix the dose with juice to disguise its taste. check heart rate; administer the dose by placing it to the back and side of the mouth. check heart rate; administer the dose by letting the infant suck it through a nipple.

not give the dose; suspect dosage error. (Digoxin is often prescribed in micrograms. Rarely is more than 1 mL administered to an infant. Because it is a potentially dangerous drug, administration guidelines are very precise. Some institutions require that digoxin dosages be checked with another professional before administration. The nurse has drawn up too much medication. These are correct procedures, but too much medication has been prepared. These are correct procedures, but too much medication has been prepared.)

Nursing care of the infant or child with congestive heart failure would include: forcing fluids appropriate to age. monitoring respirations during active periods. organizing activities to allow for uninterrupted sleep. giving larger feedings less often to conserve energy.

organizing activities to allow for uninterrupted sleep. (The child who has congestive heart failure has an excess of fluid. Monitoring vital signs is appropriate, but minimizing energy expenditure is a priority. The child needs to be well rested before feeding. The child's needs should be met as quickly as possible to minimize crying. The nurse must organize care to facilitate a decrease in his or her energy expenditure. The child often cannot tolerate larger feedings.)

Congenital heart defects have traditionally been divided into acyanotic or cyanotic defects. The nurse should recognize that in clinical practice this system is: helpful because it explains the hemodynamics involved. helpful because children with cyanotic defects are easily identified. problematic because cyanosis is rarely present in children. problematic because children with acyanotic heart defects may develop cyanosis.

problematic because children with acyanotic heart defects may develop cyanosis. (The classification does not reflect the path of blood flow within the heart. Children with cyanosis may be easily identified, but that does not help with the diagnosis. Cyanosis is present when children have defects in which oxygenated blood and unoxygenated blood are mixed. This classification is problematic. Children with traditionally named acyanotic defects may be cyanotic, and children with traditionally classified cyanotic defects may appear pink.)

After a patient returns from cardiac catheterization, the nurse assesses that the pulse distal to the catheter insertion site is weaker. The nurse should: elevate the affected extremity. record the data on the nurse's notes. notify the physician of the observation. apply warm compresses to the insertion site.

record the data on the nurse's notes. (Elevation is not necessary; the extremity is kept straight. The pulse distal to the catheter insertion site may be weaker for the first few hours after catheterization. It should gradually increase in strength. Because a weaker pulse is an expected finding, the nurse should document this and continue to monitor. The insertion site is kept dry.)

The client is an adult who has received the varicella vaccine. She has a history of arthritic-related discomfort. The most important instruction that the nurse can give this client is to avoid ________ after receiving the varicella vaccine. alcohol for 1 month dark green vegetables for 6 weeks salicylates for 6 weeks vitamin C for 1 month

salicylates for 6 weeks (Salicylates should be avoided for 6 weeks after receiving the varicella vaccine.)

A humidified atmosphere is recommended for a young child with an upper respiratory tract infection because this environment facilitates: liquefying secretions. improving oxygenation. promoting ventilation. soothing inflamed mucous membrane.

soothing inflamed mucous membrane. (The size of the droplets is too large to liquefy secretions. No additional oxygen is provided with humidified air. The humidity has no effect on ventilation. By humidifying the inspired air, the membranes inflamed by the infection and dry air are soothed.)

A young child with tetralogy of Fallot may assume a posturing position as a compensatory mechanism. The position automatically assumed by the child is: low Fowler's. prone. supine. squatting.

squatting. (Low Fowler's would assist with respiratory issues but would not assist with the need for cardiac compensation. Prone does not offer any advantage to the child. Supine does not offer any advantage to the child. The squatting or knee-chest position decreases the amount of blood returning to the heart and allows the child time to compensate.)

An early sign of congestive heart failure that the nurse should recognize is: tachypnea. bradycardia. inability to sweat. increased urine output.

tachypnea. (Tachypnea is one of the early signs that should be identified. Tachycardia at rest, dyspnea, retractions, and activity intolerance are other physical signs and symptoms. Tachycardia, not bradycardia, is one of the symptoms suggestive of congestive heart failure. The child may be diaphoretic. Urine output usually will be decreased due to decreased kidney perfusion.)

When caring for a child after a tonsillectomy, the nurse should: watch for continuous swallowing. encourage gargling to reduce discomfort. position the child on the back for sleeping. apply warm compresses to the throat.

watch for continuous swallowing. (This is the most obvious early sign of bleeding from the operative site. Gargling should be avoided because of potential trauma to the suture line. The child should be positioned on the side or abdomen to facilitate drainage. Cold is preferred. Ice collars and cold liquids are encouraged.)

A child is hospitalized with heart failure and is receiving furosemide (Lasix). Which nursing action is the priority? A. Administer oxygen. B. Encourage rest. C. Provide meticulous skin care. D. Monitor brain natriuretic peptide.

A. Administer oxygen. (A child with heart failure receiving furosemide will have pulmonary congestion from fluid backup into the lungs. The nurse should provide oxygen as the priority action. The other actions are important for this child but do not take priority. Rest will help the body heal and reduce metabolic needs. Skin care is important for edematous tissues. Brain natriuretic peptide does help quantify fluid retention, but monitoring does not actively provide care for the child.)

The student studying pediatric cardiac disorders learns that which anomalies comprise the disorder tetralogy of Fallot? (Select all that apply.) A. An overriding aorta B. Atrial septal defect (ASD) C. Hypertrophic right ventricle D. Pulmonary stenosis or atresia E. Ventricular septal defect (VSD)

A. An overriding aorta C. Hypertrophic right ventricle D. Pulmonary stenosis or atresia E. Ventricular septal defect (VSD) (The four defects seen in tetralogy of Fallot are VSD, an overriding aorta, pulmonary stenosis or atresia, and hypertrophic right ventricle. An ASD is not part of the condition.)

What is the nurse's first action when planning to teach the parents of an infant with a congenital heart defect (CHD)? Assess the parents' anxiety level and readiness to learn. Gather literature for the parents. Secure a quiet place for teaching. Discuss the plan with the nursing team.

Assess the parents' anxiety level and readiness to learn. (Any effort to organize the right environment, plan, or literature is of no use if the parents are not ready to learn or have high anxiety. Decreasing their level of anxiety is often needed before new information can be processed. A baseline assessment of prior knowledge should be taken into consideration before developing any teaching plan. Locating a quiet place for meeting with parents is appropriate; however, an assessment should be done before any teaching is done. Discussing a teaching plan with the nursing team is appropriate after an assessment of the parents' knowledge and readiness.)

An infant who was stable for a day after birth now demonstrates pallor, tachycardia, tachypnea, and circumoral cyanosis. The parent asks how the child might have a heart problem when he was stable yesterday. What information by the nurse is most accurate? A. "Blood incompatibilities can cause this problem, so we will test the mother's blood." B. "Symptoms may not appear until fetal circulation routes begin to close after birth." C. "The extra blood from the umbilical cord may have kept the baby stable for a while." D. "Your baby may have gotten an infection during birth that now is causing problems."

B. "Symptoms may not appear until fetal circulation routes begin to close after birth." (This baby has clinical manifestations of tetralogy of Fallot. While the ductus arteriosus remains patent, the infant remains stable. However, when the ductus begins closing after the first 24 hours of life, the infant's cardiovascular system becomes unstable and manifestations appear. The other statements are inaccurate.)

A nurse is assessing patients for the presence of patent ductus arteriosus (PDA). Which patient should the nurse assess first? A. 1-year old, history of frequent colds B. 4-year old, blood pressure of 102/36 mmHg C. Infant with history of poor feeding D. Toddler with murmur at right sternal border

B. 4-year old, blood pressure of 102/36 mmHg (This child has a wide pulse pressure, which is a sign of PDA. The nurse would assess this child first. Frequent colds and poor feeding can be seen in PDA, but they are vague symptoms and could be related to a number of other conditions. The murmur of a PDA is heard best at the left subclavicular margin.)

A child has a large ventricular septal defect (VSD) with left-to-right shunting. Which information about VSDs does the faculty member explain to the nursing student? (Select all that apply.) A. The child will have obvious cyanosis. B. A harsh, pansystolic murmur is heard. C. This is the most common congenital heart defect. D. Some VSDs can close spontaneously. E. Transcatheter closure is common.

B. A harsh, pansystolic murmur is heard. C. This is the most common congenital heart defect. D. Some VSDs can close spontaneously. (A VSD is the most common congenital heart defect and is one of the mildest forms. The murmur of a VSD is harsh, pansystolic, and heard best at the lower left sternal border. Some VSDs can close spontaneously; those that need invasive closure must be surgically repaired. Presently, transcatheter closure is not available.)

A student nurse asks the faculty why a child with patent ductus arteriosus (PDA) is taking a nonsteroidal anti-inflammatory drug (NSAID). Which response by the faculty is the most appropriate? A. Decreases venous stasis, lowering risks of clotting B. Inhibits prostaglandin, which helps close the PDA C. Provides long-lasting pain and inflammation control D. Reduces swelling around the PDA, making surgery easier

B. Inhibits prostaglandin, which helps close the PDA (Prostaglandin helps keep the PDA open, so an NSAID that inhibits prostaglandin synthesis will help close the opening. This is especially beneficial for premature infants. It is not used for venous stasis, pain relief, or swelling.)

An infant is discharged after open heart surgery. The infant is going home on oxygen and with multiple medications. Which instruction by the nurse is the priority? A. "Be sure to keep all postoperative appointments." B. "Do not allow anyone to pick up or hold the baby." C. "If your baby is irritable, check oxygen saturation." D. "Monitor the incision for redness or warmth."

C. "If your baby is irritable, check oxygen saturation." (A child going home on oxygen will have an oximeter. This child's parents need to be educated on signs of heart failure (including decreased oxygenation), including irritability in a small child. They should be told to check the oxygen saturation when irritability is seen. Keeping appointments and monitoring a surgical incision are appropriate instructions for any postoperative patient. The baby can be picked up and held but must be cradled and not picked up under the arms until the sternum has healed.)

An infant is experiencing dyspnea related to patent ductus arteriosus (PDA). What does the nurse understand regarding why dyspnea occurs? a. Blood is circulated through the lungs again, causing pulmonary circulatory congestion. b. Blood is shunted past the pulmonary circulation, causing pulmonary hypoxia. c. Blood is shunted past cardiac arteries, causing myocardial hypoxia. d. Blood is circulated through the ductus from the pulmonary artery to the aorta, bypassing the left side of the heart.

a. Blood is circulated through the lungs again, causing pulmonary circulatory congestion. (When PDA is present, oxygenated blood recycles through the lungs, overburdening the pulmonary circulation.)

What does the nurse explain that a ventricular septal defect will allow? a. Blood to shunt left to right, causing increased pulmonary flow and no cyanosis b. Blood to shunt right to left, causing decreased pulmonary flow and cyanosis c. No shunting because of high pressure in the left ventricle d. Increased pressure in the left atrium, impeding circulation of oxygenated blood in the circulating volume

a. Blood to shunt left to right, causing increased pulmonary flow and no cyanosis (Pulmonary blood flow is increased when a ventricular septal defect exists. The blood shifts from left to right because of the higher pressure in the left ventricle. This particular shift does not cause cyanosis.)

What is the most important action for the nurse to perform before administering a patient's daily dose of digoxin (Lanoxin)? a. Check the patient's apical pulse for a full 60 seconds. b. Assess the patient's dependent body areas for edema formation. c. Ask whether the patient has experienced any heart palpitations during the last 24 hours. d. Verify that the time of administration today is within one-half hour of the time the drug was administered yesterday.

a. Check the patient's apical pulse for a full 60 seconds. (Although all the actions are important, the most important is to ensure that the pulse rate is between 60 and 100 beats per minute and is regular before administering digoxin or any other cardiac glycoside. For an irregular heart rate or one that is outside of this range, the dose must be held and the prescriber notified immediately.)

How would the nurse caring for an infant with congestive heart failure (CHF) modify feeding techniques to adapt for the child's weakness and fatigue? (Select all that apply.) a. Feeding more frequently with smaller feedings b. Using a soft nipple with enlarged holes c. Holding and cuddling the child during feeding d. Substituting glucose water for formula e. Offering high-caloric formula

a. Feeding more frequently with smaller feedings b. Using a soft nipple with enlarged holes c. Holding and cuddling the child during feeding e. Offering high-caloric formula (Infants with CHF fatigue easily. Feeding can be given more frequently in smaller amounts through a soft, large-holed nipple. Formulas with a denser caloric content can be offered. The child may be encouraged to nurse if he or she is held.)

The nurse is preparing to administer a dose of digoxin (Lanoxin) to a child in heart failure (HF). Which is a beneficial effect of administering digoxin (Lanoxin)? a. It decreases edema. b. It decreases cardiac output. c. It increases heart size. d. It increases venous pressure.

a. It decreases edema. (Digoxin has a rapid onset and is useful for increasing cardiac output, decreasing venous pressure, and, as a result, decreasing edema. Cardiac output is increased by digoxin. Heart size and venous pressure are decreased by digoxin.)

The nurse provides teaching for a patient who will begin taking indomethacin (Inderal) to treat rheumatoid arthritis. Which statement by the patient indicates a need for further teaching? a. "I should limit sodium intake while taking this drug." b. "I should take indomethacin on an empty stomach." c. "I will need to check my blood pressure frequently." d. "I will take the medication twice daily."

b. "I should take indomethacin on an empty stomach." (Indomethacin is very irritating to the stomach and should be taken with food. It can cause sodium retention and elevated blood pressure, so patients should limit sodium intake. The medication is taken twice daily.)

An 8-year-old child is receiving digoxin (Lanoxin). The nurse should notify the practitioner and withhold the medication if the apical pulse is less than _____ beats/min. a. 60 b. 70 c. 90 d. 100

b. 70 (If a 1-minute apical pulse is less than 70 beats/min for an older child, the digoxin is withheld; 60 beats/min is the cut-off for holding the digoxin dose in an adult. A pulse below 90 to 110 beats/min is the determination for not giving a digoxin dose to infants and young children.)

A child is diagnosed with iron deficiency anemia. What will the nurse explain can occur if this disorder goes untreated? a. Hemorrhage b. Heart failure c. Infection d. Pulmonary embolism

b. Heart failure (Untreated iron deficiency anemias progress slowly, and in severe cases the heart muscle becomes too weak to function. If this happens, heart failure follows.)

A patient is receiving digoxin 0.25 mg/day. What should the nurse do prior to administering this medication? a. Count an apical pulse for 15 seconds. b. Hold the dose if the apical rate is 57 beats/min. c. Give the dose if the apical rate is 59 beats/min. d. Double the dose if the rate is 62 beats/min.

b. Hold the dose if the apical rate is 57 beats/min. (The dose should be held if the apical rate is less than 60 beats/min for 1 minute.)

A patient taking digoxin (Lanoxin) has all of the following laboratory blood values. Which value does the nurse report to the prescriber before administering the next dose of digoxin? a. Sodium 133 mEq/L b. Potassium 2.8 mEq/L c. Blood urea nitrogen 9 mg/dL d. White blood cell count 11,000 cells/mm3

b. Potassium 2.8 mEq/L (Serum potassium level affects the activity of digoxin. A value of 2.8 is low (hypokalemia). Any abnormal potassium value (high or low) requires the prescriber to change the digoxin dosage. In addition, action is needed to bring this critical electrolyte value back to its normal range. Although the other laboratory values are slightly abnormal, none are critically abnormal or likely to have an effect on digoxin activity.)

Which is the most appropriate nursing action related to the administration of digoxin (Lanoxin) to an infant? a. Counting the apical rate for 30 seconds before administering the medication b. Withholding a dose if the apical heart rate is less than 100 beats/min c. Repeating a dose if the child vomits within 30 minutes of the previous dose d. Checking respiratory rate and blood pressure before each dose

b. Withholding a dose if the apical heart rate is less than 100 beats/min (As a rule, if the pulse rate of an infant is below 100 beats/min, the medication is withheld and the physician is notified.)

A patient with heart failure is prescribed digoxin (Lanoxin). The patient reports nausea, blurry vision, and feeling like the heart is skipping beats. What is the nurse's best action? a. Administer the scheduled dose because it will correct these side effects. b. Assess the patient's heart rate and then administer the scheduled dose. c. Hold the dose and notify the prescriber immediately. d. Place the patient on complete bed rest.

c. Hold the dose and notify the prescriber immediately. (Nausea, blurry vision, and heart rate or rhythm changes are signs of digoxin overdose. The dose should be held and the prescriber notified. Most likely the nurse will also obtain a blood sample to measure the digoxin level. Digoxin has a very narrow therapeutic range (0.8 to 2 ng/mL) and this level should be checked whenever a patient shows any signs of toxicity. Symptoms usually resolve when the drug is held and the body eliminates it.)

Which of the following is a clinical manifestation of the systemic venous congestion that can occur with congestive heart failure? a. Tachypnea b. Tachycardia c. Peripheral edema d. Pale, cool extremities

c. Peripheral edema (Peripheral edema, especially periorbital edema, is a clinical manifestation of systemic venous congestion.)

A patient asks the nurse how prescribed digoxin (Lanoxin) will help improve heart failure. What is the nurse's best response? a. "It dilates your arteries and decreases your blood pressure." b. "It directly perfuses your kidneys which decreases your fluid volume and blood pressure." c. "It increases your heart rate which will result in improved cardiac output." d. "It increases the force of heart contractions and slows down the heart rate to improve cardiac output."

d. "It increases the force of heart contractions and slows down the heart rate to improve cardiac output." (Digoxin is a cardiac glycoside drug that works on the muscle fiber of the heart to increase the force of each contraction. It also slows down the heart rate. These actions improve cardiac output.)

The perinatal nurse contacts the pediatrician about a heart murmur that was auscultated during a routine newborn assessment. This finding would be abnormal at: a. 8 to 12 hours b. 12 to 24 hours c. 24 to 48 hours d. 48 to 72 hours

d. 48 to 72 hours (It is not uncommon to hear murmurs in infants less than 24 hours old. The murmurs are characterized by a sound (best heard near the sternal border at the second or third intercostal space on the left side) that grows louder during systole. Although a heart sound arising from a patent ductus arteriosus may be heard initially, the sound disappears within 2 to 3 days when the ductus closes. If a murmur remains audible after the second day of life and intensifies to a "whoosh" sound, further investigation is warranted because this finding is not characteristic of a patent ductus and may indicate the presence of another type of heart lesion.)

The parent of a 1-year-old child with tetralogy of Fallot asks the nurse, "Why do my child's fingertips look like that?" On what understanding does the nurse base a response? a. Clubbing occurs as a result of untreated congestive heart failure. b. Clubbing occurs as a result of a left-to-right shunting of blood. c. Clubbing occurs as a result of decreased cardiac output. d. Clubbing occurs as a result of chronic hypoxia.

d. Clubbing occurs as a result of chronic hypoxia. (Clubbing of the fingers develops in response to chronic hypoxia.)

The parents of a young child with congestive heart failure tell the nurse that they are "nervous" about giving digoxin. The nurse's response should be based on which of the following? a. It is a safe, frequently used drug. b. It is difficult to either overmedicate or undermedicate with digoxin. c. Parents lack the expertise necessary to administer digoxin. d. Parents must learn specific, important guidelines for administration of digoxin.

d. Parents must learn specific, important guidelines for administration of digoxin. (Digoxin has a narrow therapeutic range. The margin of safety between therapeutic, toxic, and lethal doses is very small. Specific guidelines are available for parents to learn how to administer the drug safely and to monitor for side effects.)

An infant with congestive heart failure is receiving digoxin (Lanoxin). What does the nurse recognize as a sign of digoxin toxicity? a. Restlessness b. Decreased respiratory rate c. Increased urinary output d. Vomiting

d. Vomiting (Symptoms of digoxin toxicity include: nausea, vomiting, anorexia, irregularity in pulse rate and rhythm, and a sudden change in pulse.)


Ensembles d'études connexes

Chapter 6 - Interest Rate Parity (and part of chapter 7 - speculation and risk in foreign exchange market)

View Set

Pharmacology: Chapter 24 - Adrenergic Drugs

View Set

Macroeconomics Chapters 15, 16, 17, & 18

View Set

UNIT 6 STUDY GUIDE (Career Management)

View Set

Anatomy and Physiology Chapter One

View Set

Autonomic Nervous System and Central Nervous System

View Set

Financial Management Midterm Review

View Set

Chapter 7: Improving Decisions with Marketing Information

View Set